K
Khách

Hãy nhập câu hỏi của bạn vào đây, nếu là tài khoản VIP, bạn sẽ được ưu tiên trả lời.

Sửa đề:  Cho a, b, c là các số thực dương thỏa mãn điều kiện abc=1. Chứng minh rằng

\(\frac{1}{ab+b+2}+\frac{1}{bc+c+2}+\frac{1}{ca+a+2}\le\frac{3}{4}\)

Áp dụng bđt Cauchy-Schwarz ta có:

\(\frac{1}{ab+b+2}=\frac{1}{ab+1+b+1}\le\frac{1}{4}\left(\frac{1}{ab+1}+\frac{1}{b+1}\right)\) \(=\frac{1}{4}\left(\frac{abc}{ab\left(1+c\right)}+\frac{1}{b+1}\right)=\frac{1}{4}\left(\frac{c}{1+c}+\frac{1}{b+1}\right)\)

Tương tự \(\frac{1}{bc+c+2}\le\frac{1}{4}\left(\frac{a}{a+1}+\frac{1}{c+1}\right)\)

          \(\frac{1}{ca+a+2}\le\frac{1}{4}\left(\frac{b}{b+1}+\frac{1}{a+1}\right)\)

Cộng từng vế các bđt trên ta được

\(VT\le\frac{1}{4}\left(\frac{a+1}{a+1}+\frac{b+1}{b+1}+\frac{c+1}{c+1}\right)=\frac{3}{4}\)

Vậy bđt được chứng minh

Dấu "=" xảy ra khi a=b=c=1

21 tháng 12 2019

\(\frac{a}{1+b^2}=\frac{a\left(1+b^2\right)-ab^2}{1+b^2}=a-\frac{ab^2}{1+b^2}\ge a-\frac{ab^2}{2b}=a-\frac{ab}{2}\)

Tương tự:

\(\frac{b}{1+c^2}\ge b-\frac{bc}{2};\frac{c}{1+a^2}\ge c-\frac{ca}{2}\)

Cộng lại:

\(\frac{a}{1+b^2}+\frac{b}{1+c^2}+\frac{c}{1+a^2}\ge a+b+c-\frac{ab}{2}-\frac{bc}{2}-\frac{ca}{2}\)

\(\Rightarrow VT\ge a+b+c\)

Mặt khác:

\(\frac{9}{a+b+c}\le\frac{1}{a}+\frac{1}{b}+\frac{1}{c}\le3\Rightarrow9\le3\left(a+b+c\right)\Rightarrow a+b+c\ge3\)

Khi đó:

\(VT\ge a+b+c\ge3\left(đpcm\right)\)

Dấu "=" xảy ra tại \(a=b=c=1\)

15 tháng 4 2020

????????

bđt phụ sai mà cũng ko đc chuẩn hóa

23 tháng 8 2017

\(\frac{ab}{a^2+b^2}\le\frac{ab}{2ab}=\frac{1}{2}\)

tương tự \(\frac{\Rightarrow ab}{a^2+b^2}+\frac{bc}{b^2+c^2}+\frac{ac}{a^2+c^2}\le\frac{3}{2}\)

=>Thắng Nguyễn :cm theo cách đó sai

30 tháng 6 2017

Bunhia thì phải hoặc tương đương thần chưởng @@
Có lẽ bunhia đấy :vv

30 tháng 6 2017

Câu này t dùng vi-et giải được. Nhưng để mai đi. Giờ giải bằng điện thoại thì khó quá

10 tháng 12 2017

Ta có \(ab+bc+ca\ge3\sqrt[3]{a^2b^2c^2}\)\(\Rightarrow3\sqrt[3]{a^2b^2c^2}\le3\Leftrightarrow abc\le1\)

\(\Rightarrow\)\(\frac{1}{1+a^2\left(b+c\right)}\le\frac{1}{abc+a^2\left(b+c\right)}\)\(=\frac{1}{a\left(ab+bc+ca\right)}=\frac{1}{3a}\)

\(CMTT\Rightarrow\frac{1}{1+b^2\left(c+a\right)}\le\frac{1}{3b}\)

                  \(\frac{1}{1+c^2\left(a+b\right)}\le\frac{1}{3c}\)

\(\Rightarrow VT\le\frac{1}{3a}+\frac{1}{3b}+\frac{1}{3c}\)\(=\frac{ab+bc+ca}{3abc}=\frac{1}{abc}\)

4 tháng 11 2019

CM BĐT : \(\left(x^2+y^2+z^2\right)^2\ge3\left(x^3y+y^3z+z^3x\right)\)   ( * )

\(\frac{a}{ab+1}=\frac{a\left(ab+1\right)-a^2b}{ab+1}=a-\frac{a^2b}{ab+1}\)

TT ....

Áp dụng BĐT ( * ) với x = \(\sqrt{a}\); y = \(\sqrt{b}\); z = \(\sqrt{c}\) vào bài toán, ta có :

\(\frac{a}{ab+1}+\frac{b}{bc+1}+\frac{c}{ca+1}=a+b+c-\frac{a^2b}{ab+1}-\frac{b^2c}{bc+1}-\frac{c^2a}{ac+1}\)

\(\ge3-\frac{a^2b}{2\sqrt{ab}}-\frac{b^2c}{2\sqrt{bc}}-\frac{c^2a}{2\sqrt{ac}}=3-\frac{\sqrt{a^3b}+\sqrt{b^3c}+\sqrt{c^3a}}{2}\ge3-\frac{\frac{\left(a+b+c\right)^2}{3}}{2}=\frac{3}{2}\)

Dấu " = " xảy ra \(\Leftrightarrow a=b=c=1\)